Középiskolai Matematikai és Fizikai Lapok
Informatika rovattal
Kiadja a MATFUND Alapítvány
Már regisztráltál?
Új vendég vagy?

Fórum: Érdekes matekfeladatok

  [1]    [2]    [3]    [4]    [5]    [6]    [7]    [8]    [9]    [10]    [11]    [12]    [13]    [14]    [15]    [16]    [17]    [18]    [19]    [20]    [21]    [22]    [23]    [24]    [25]    [26]    [27]    [28]    [29]    [30]    [31]    [32]    [33]    [34]    [35]    [36]    [37]    [38]    [39]    [40]    [41]    [42]    [43]    [44]    [45]    [46]    [47]    [48]    [49]    [50]    [51]    [52]    [53]    [54]    [55]    [56]    [57]    [58]    [59]    [60]    [61]    [62]    [63]    [64]    [65]    [66]    [67]    [68]    [69]    [70]    [71]    [72]    [73]    [74]    [75]    [76]    [77]    [78]    [79]    [80]    [81]    [82]    [83]    [84]    [85]    [86]    [87]    [88]    [89]    [90]    [91]    [92]    [93]    [94]    [95]    [96]    [97]    [98]    [99]    [100]    [101]    [102]    [103]    [104]    [105]    [106]    [107]    [108]    [109]    [110]    [111]    [112]    [113]    [114]    [115]    [116]    [117]    [118]    [119]    [120]    [121]    [122]    [123]    [124]    [125]    [126]    [127]    [128]    [129]    [130]    [131]    [132]    [133]    [134]    [135]    [136]    [137]    [138]    [139]    [140]    [141]    [142]    [143]    [144]    [145]    [146]    [147]    [148]    [149]    [150]    [151]    [152]    [153]    [154]    [155]    [156]    [157]    [158]    [159]    [160]    [161]  

Szeretnél hozzászólni? Jelentkezz be.
[1533] nadorp2006-11-23 00:10:27

p páratlan prím

Előzmény: [1532] nadorp, 2006-11-23 00:06:34
[1532] nadorp2006-11-23 00:06:34

Ha n=p2 ahol p prím, akkor \frac{n}{\phi(n)}=\frac{p}{p-1} nem egész és a \sum_{k=1}^\infty\frac{p_k-1}{p_k} sor divergens, hiszen az általános tag nem tart 0-ba

Előzmény: [1528] S.Ákos, 2006-11-22 20:00:39
[1531] Sirpi2006-11-22 22:53:47

Lehet hogy rosszul gondolom, de én az állítást úgy értelmeztem, hogy azon egészek reciprokösszege, melyekre teljesül, hogy nem osztja őket a nála kisebb relatív prímek száma, végtelen.

Előzmény: [1529] Lóczi Lajos, 2006-11-22 22:26:15
[1530] nadorp2006-11-22 22:41:06

Egy olyan permutáció, amely pontosan két elemet felcserél, a többit békén hagyja.

Előzmény: [1526] hobbymatekos, 2006-11-21 14:47:07
[1529] Lóczi Lajos2006-11-22 22:26:15

Csak 4 dolgot nem értek:

1. mi a különbség hi és Hi között

2. mit jelöl a \varphi függvény

3. mit jelöl a kapcsoszárójel

4. mire vonatkozik a limesz, ugyanis a szummában nincs már szabad változó

(Jobban érteném hi jelentését, ha látnám az első pár tagját.)

Előzmény: [1528] S.Ákos, 2006-11-22 20:00:39
[1528] S.Ákos2006-11-22 20:00:39

remélem még nem szerepelt: tekintsük azokat az Hn pozitív egész számokat, melyre 0<\{\frac{n}{\varphi(n)}\}! Legyenek ezek a számok h1;h2;.... bizonyítandó, hogy lim{\sum_{i=1}^\infty\frac{1}{h_i}=\infty}!

[1527] Lóczi Lajos2006-11-21 21:36:41

Nos igen, és a többit elemi képlettel nem is tudjuk felírni; mindenesetre a feladat néhány ártatlan probléma között kakukktojásként bújt meg...

Előzmény: [1525] jenei.attila, 2006-11-21 13:55:19
[1526] hobbymatekos2006-11-21 14:47:07

Szia. Mi a transzpozició definiciója?

Előzmény: [1520] Cckek, 2006-11-18 11:59:12
[1525] jenei.attila2006-11-21 13:55:19

Sajnos ennek az egyenletnek egy perióduson belül 10 megoldása van, ebből 2-őt találtál meg.

Előzmény: [1524] ágica, 2006-11-20 23:03:50
[1524] ágica2006-11-20 23:03:50

Ha abból indulunk ki, hogy a \frac{\sqrt{3}}{4}-et \frac{1}{2} és \frac{\sqrt{3}}{2} szorzatára bontjuk, akkor négy eset lehetséges: ha sin x értéke \frac{1}{2} és cos 8x értéke \frac{\sqrt{3}}{2}, ugyanez fordítva, illetve ugyanezek negatív előjelekkel. Ebből egyedül az teljesülhet, ha \sin{x}=-\frac{\sqrt{3}}{2} és \cos{8x}=-\frac{1}{2}, ekkor x=-\frac{\pi}{3}+k2\pi vagy x=-\frac{2\pi}{3}+k2\pi. Ezek megoldások tehát, hogy van-e más is, azt nem tudom. :)

Előzmény: [1521] Lóczi Lajos, 2006-11-19 17:44:40
[1523] Lóczi Lajos2006-11-20 02:13:05

Nem talált. :)

Előzmény: [1522] Cckek, 2006-11-19 23:00:14
[1522] Cckek2006-11-19 23:00:14

x=\frac{\pi}{3}+2k\pi ???

Előzmény: [1521] Lóczi Lajos, 2006-11-19 17:44:40
[1521] Lóczi Lajos2006-11-19 17:44:40

Oldjuk meg a valós számok halmazán a

\sin(x)\cos(8x)=\frac{\sqrt{3}}{4}

trigonometrikus egyenletet.

[1520] Cckek2006-11-18 11:59:12

Még valami. Az vajon eldönthető-e, hogy egy k inverzióval rendelkező n-edrendű permutáció hányféle képpen bontható fel transzpozíciók szorzatára? Én ezzel probálkoztam, persze nem jutottam sehova.:)

[1519] Cckek2006-11-18 11:51:16

Nagyon érdekes okfejtés. Az nyílvánvaló, már csak az inverziók számolási módszeréből is, hogy a t1+t2+...+tn-1=k összeg összes lehetséges képzésének a száma adja meg az n-edrendű k inverzióval rendelkező permutációk számát, hiszen ha a permutáció alsó sorában 1 előtt t1, 1-nél nagyobb, 2 előtt t2, 2-nél nagyobb,.. (n-1) előtt tn-1, (n-1)-nél nagyobb elem van akkor a permutáció inverzióinak a száma t1+t2+...+tn-1. Az is látható, hogy 0\let1\le(n-1), 0\let2\le(n-2),...,0\letn-1\le1 Tehát, a problémát, kitűnően letárgyáltátok, itt valóban nem beszélhetünk zárt alakról, annyit még hozzátennék, hogy amennyiben Skn-el jelöljük a k inverzióval rendelkező n-edrendű permutációk számát úgy fennáll :

S_k^{n+1}=\sum_{i=0}^k{S_i^n}

Attila is említett egy rekurziót. Az hogy néz ki?

Előzmény: [1518] nadorp, 2006-11-18 10:22:10
[1518] nadorp2006-11-18 10:22:10

Megpróbálom vázolni a gondolatmenetemet. Jelölje az n-edrendű k inverziót tartalmazó permutációk számát I(n,k) és legyen p(n,k) egy ilyen permutáció. Ekkor ha az n+1 elemet a permutáció végére tesszük, akkor egy p(n+1,k) permutációt kapunk. Ha az n+1-et eggyel balra tolva a n-dik helyre tesszük, akkor egy p(n+1,k+1) permutációt kapunk stb. ha az n+1 az első helyen van, akkor egy p(n+1,k+n) permutációt kapunk. Foglaljuk be ezt az alábbi táblázatba, ahol a k-dik oszlopban a p(n,k)-ból kapható permutációk találhatóak.

\matrix{p(n+1,0) & p(n+1,1) & ... & p\left(n+1,\frac{n(n-1)}2\right) \cr
        p(n+1,1) & p(n+1,2) & ... & p\left(n+1,\frac{n(n-1)}2+1\right) \cr
        ... \cr
        p(n+1,n) & p(n+1,n+1) & ... & p\left(n+1,\frac{n(n-1)}2+n\right)}

Tekintsük még a következő hasonló táblázatot

\matrix{I(n,0) & I(n,1) & ... & I\left(n,\frac{n(n-1)}2\right) \cr
        I(n,0) & I(n,1) & ... & I\left(n,\frac{n(n-1)}2\right) \cr
        ... \cr
        I(n,0) & I(n,1) & ... & I\left(n,\frac{n(n-1)}2\right)}

Nyilvánvaló, hogy I(n+1,k) úgy kapható meg, hogy ahol az első táblázatban p(n+1,k) van, összeadjuk a második táblázatban az ezeken a helyeken szerplő I(n,...) értékeket.Tehát

I(n+1,0)=I(n,0)

I(n+1,1)=I(n,0)+I(n,1)

I(n+1,2)=I(n,0)+I(n,1)+I(n,2) stb..

Ezek az összegek éppen balról jobbra haladva a "balról jobbra fel átlókon" levő elemek összege. Ha most fn(x) az I(n,k) sorozat (n fix) generátor függvénye, azaz

f_n(x)=a_0+a_1x+a_2x^2+...+a_{\frac{n(n-1)}2}x^{\frac{n(n-1)}2}, ahol ak=I(n,k), akkor

f_{n+1}(x)=a_0+(a_0+a_1)x+(a_0+a_1+a_2)x^2+...+a_{\frac{n(n-1)}2}x^{\frac{n(n+1)}2}=f_n(x)(1+x+x^2+...+x^n)

Előzmény: [1516] jenei.attila, 2006-11-16 15:44:31
[1517] jenei.attila2006-11-16 15:50:44

Egyébként (mint azóta utána néztem), D.E. Knuth: A számítógépes programozás művészete c. könyv harmadik kötete (Rendezések és keresések) is részletesen foglalkozik a problémával. Ott egy érdekes rekurziót is megad. A könyvet nagyon ajánlom mindenkinek.

Előzmény: [1516] jenei.attila, 2006-11-16 15:44:31
[1516] jenei.attila2006-11-16 15:44:31

Nadorpnak, ha jól tudom van egy ettől különböző megoldása. Engem is érdekelne, úgyhogy Nadorp! Légyszíves oszd meg velünk, Te mire jutottál. Köszi.

Előzmény: [1515] Cckek, 2006-11-16 15:40:11
[1515] Cckek2006-11-16 15:40:11

Nos, ilyen messzire nem jutottam, és azt sem tudtam, hogy ezzzel már foglalkoztak, épp csak felvetődött bennem.:)

Előzmény: [1514] jenei.attila, 2006-11-15 19:34:03
[1514] jenei.attila2006-11-15 19:34:03

Elírtam: "permutációban hány előző elem kisebb" helyett: permutációban hány előző elem nagyobb

Előzmény: [1513] jenei.attila, 2006-11-15 19:27:32
[1513] jenei.attila2006-11-15 19:27:32

A feladat megtalálható Lovász László Kombinatorikai problémák és feladatok c. könyvében. Itt a megoldást az ún. inverzió vektorok használatával adják meg. Egy n-ed rendű p permutáció inverzió vektora egy n elemű t vektor (tömb), amelynek j-edik eleme azt adja meg, hogy a permutációban hány előző elem kisebb, mint a j-edik elem: tj=|1<=i<j:p(i)>p(j)|, vagyis rögzített j mellet a kisebb indexekkel képezhető inverziók számát. Világos, hogy 0<=tj<=j-1, és a t elemeinek összege éppen a p inverzióinak számát adja. Fordítva is igaz: egy feltételeknek megfelelő t vektor egyértelműen meghatározza a p permutációt (ez is egy érdekes kis feladat). Ezért a k=t1,+t2+...+tn egyenlet feltételnek megfelelő (0<=tj<=j-1) megoldásait kell összeszámolni, vagyis k hányféleképpen írható fel ilyen módon n db. szám összegeként. Ennek az ún. partíciós problémának a megoldását adja a Nadorp által felírt polinom k-ad fokú tagjának együtthatója.

Erre a megoldásra gondoltál?

Előzmény: [1511] Cckek, 2006-11-14 19:03:42
[1512] nadorp2006-11-15 17:03:41

Félek tőle, hogy erre nincs zárt formula. Ha tévedtem, akkor annál jobb. Az az erős sejtésem ( bár ez is kétséges, úgyhogy ne dobáljatok meg kővel), hogy ha a keresett számot I(n,k)-val jelöljük, akkor az

fn(x)=(1+x)(1+x+x2)...(1+x+...xn-1) polinomban xk együtthatója éppen I(n,k)

Előzmény: [1511] Cckek, 2006-11-14 19:03:42
[1511] Cckek2006-11-14 19:03:42

Jó, részemről befejezve felejtsük el:)) Itt van egy megint csak érdekes feladat:)) Hány n-edrendű permutáció inverzióinak a száma k???

[1510] jenei.attila2006-11-14 16:29:21

Egyáltalán nem akartalak megsérteni, szerintem nem is volt rá okod, hogy megsértődj. A véleményemet pedig hadd mondjam már el, én teljesen jószándékúan kérdeztem, hogy mi a feladatban az érdekesség, trükk. Most, hogy már megvilágítottad a feladat hátterét, valóban érdekesebbnek találom, bár még mindíg kicsit "megcsinált" ízű. Egyébként engedd meg, hogy azzal a feladattal foglalkozzak, amelyik nekem tetszik, a versengéssel kapcsolatban meg nem értem mire gondolsz.

Előzmény: [1509] Cckek, 2006-11-14 15:49:20
[1509] Cckek2006-11-14 15:49:20

Kedves Attila. Eddig mintha starpás és mesterkélt lett volna. hmm:) Amúgy ez csak egy feladat, kitűztem mert reméltem, hogy van akinek örömet okoz a megoldása. Ha neked nem, hát akkor ne foglalkozz vele, ez itt nem verseny, és remélem még csak versengés sem.

Előzmény: [1504] jenei.attila, 2006-11-14 11:32:30

  [1]    [2]    [3]    [4]    [5]    [6]    [7]    [8]    [9]    [10]    [11]    [12]    [13]    [14]    [15]    [16]    [17]    [18]    [19]    [20]    [21]    [22]    [23]    [24]    [25]    [26]    [27]    [28]    [29]    [30]    [31]    [32]    [33]    [34]    [35]    [36]    [37]    [38]    [39]    [40]    [41]    [42]    [43]    [44]    [45]    [46]    [47]    [48]    [49]    [50]    [51]    [52]    [53]    [54]    [55]    [56]    [57]    [58]    [59]    [60]    [61]    [62]    [63]    [64]    [65]    [66]    [67]    [68]    [69]    [70]    [71]    [72]    [73]    [74]    [75]    [76]    [77]    [78]    [79]    [80]    [81]    [82]    [83]    [84]    [85]    [86]    [87]    [88]    [89]    [90]    [91]    [92]    [93]    [94]    [95]    [96]    [97]    [98]    [99]    [100]    [101]    [102]    [103]    [104]    [105]    [106]    [107]    [108]    [109]    [110]    [111]    [112]    [113]    [114]    [115]    [116]    [117]    [118]    [119]    [120]    [121]    [122]    [123]    [124]    [125]    [126]    [127]    [128]    [129]    [130]    [131]    [132]    [133]    [134]    [135]    [136]    [137]    [138]    [139]    [140]    [141]    [142]    [143]    [144]    [145]    [146]    [147]    [148]    [149]    [150]    [151]    [152]    [153]    [154]    [155]    [156]    [157]    [158]    [159]    [160]    [161]